Όρια με ολοκληρώματα

Συντονιστές: grigkost, Κοτρώνης Αναστάσιος

Άβαταρ μέλους
Κοτρώνης Αναστάσιος
Επιμελητής
Δημοσιεύσεις: 3203
Εγγραφή: Κυρ Φεβ 22, 2009 11:11 pm
Τοποθεσία: Μπροστά στο πισί...
Επικοινωνία:

Re: Όρια με ολοκληρώματα

#161

Μη αναγνωσμένη δημοσίευση από Κοτρώνης Αναστάσιος » Σάβ Μαρ 19, 2011 12:16 am

erxmer έγραψε:66) Ας υπολογιστεί το \boxed{\displaystyle{\lim_{n \to +\infty}n^{-2}\underbrace{\sum_{i=1}^{n}{\sum_{j=1}^{n^2}{\frac{1}{\sqrt{n^2+ni+j}}}}}_{:=A_{n}}}}
Χρησιμοποιούμε το ολοκληρωτικό κριτήριο σύγκλισης:

Αν η f:[N,+\infty)\to\mathbb R είναι γνησίως φθίνουσα όπου N ακέραιος, τότε \displaystyle{\int_{N}^{M+1}f(x)\,dx\leq\sum_{k=N}^{M}f(k)\leq f(N)+\int_{N}^{M}f(x)\,dx} για κάθε \mathbb Z\ni M>N.

Έστω n σταθερό μεγάλο.
Εκτιμούμε αρχικά το δεύτερο άθροισμα για i σταθερό. Η (n^2+ni+x)^{-1/2} είναι γνησίως φθίνουσα στο [1,+\infty), άρα για (M,N)=(n^2,1) στο κριτήριο παίρνουμε

\displaystyle{2\left((2n^2+ni+1)^{1/2}-(n^2+ni+1)^{1/2}\right)=\int_{1}^{n^2+1}(n^2+ni+x)^{-1/2}\,dx\leq\sum_{j=1}^{n^2}(n^2+ni+j)^{-1/2}\leq}

\displaystyle{(n^2+ni+1)^{-1/2}+\int_{1}^{n^2}((n^2+ni+x))^{-1/2}\,dx=2\left((2n^2+ni)^{-1/2}-(n^2+ni+1)^{-1/2}\right)}.

Αθροίζουμε τις ανισότητες για i=1,\ldots,n και παίρνουμε

\displaystyle{2\sum_{i=1}^{n}\left((2n^2+ni+1)^{1/2}-(n^2+ni+1)^{1/2}\right)\leq A_{n}\leq\sum_{i=1}^{n}(n^2+ni+1)^{-1/2}+2\sum_{i=1}^{n}\left((2n^2+ni)^{-1/2}-(n^2+ni+1)^{-1/2}\right)}.

Εφαρμόζουμε μια φορά ακόμα το κριτήριο για τις συναρτήσεις

f_{1}(x):=(n^2+nx+1)^{-1/2}, f_{2}(x):=(2n^2+nx)^{-1/2}-(n^2+nx+1)^{-1/2} στο [1,n] για να πάρουμε το δεξί φράγμα και για τη συνάρτηση

f_{3}(x):=(2n^2+nx+1)^{1/2}-(n^2+nx+1)^{1/2} στο ίδιο διάστημα για το αριστερό φράγμα, πολλαπλασιάζουμε με 1/n^2, παίρνουμε όρια και από παρεμβολή n^{-2}A_{n}\to\frac{4}{3}-\frac{16\sqrt{2}}{3}+4\sqrt{3}.

Το παραπάνω κριτήριο είναι πάρα πολύ χρήσιμο γιατί μπορούμε να περνάμε από άθροισμα σε ολοκλήρωμα που έχουμε μεγαλύτερη ελευθερία κινήσεων. Αν πχ κάπου σε μια παράσταση που ψάχνουμε όριο παρουσιάζεται μια σειρά \sum_{k=1}^{n}f(n) που αποκλίνει, από το κριτήριο ξέρουμε ότι \sum_{k=1}^{n}f(k)\stackrel{n\to+\infty}{\sim}\int_{1}^{n}f(x)\,dx, και το ολοκλήρωμα μπορεί ενδεχομένως να υπολογίζεται ή να μπορούμε ευκολότερα να εκτιμήσουμε τη συμπεριφορά του με ολοκλήρωση κατά μέρη ή άλλους τρόπους. Αυτά...
τελευταία επεξεργασία από Κοτρώνης Αναστάσιος σε Κυρ Απρ 03, 2011 2:45 pm, έχει επεξεργασθεί 1 φορά συνολικά.


Εσύ....; Θα γίνεις κανίβαλος....;
Άβαταρ μέλους
Κοτρώνης Αναστάσιος
Επιμελητής
Δημοσιεύσεις: 3203
Εγγραφή: Κυρ Φεβ 22, 2009 11:11 pm
Τοποθεσία: Μπροστά στο πισί...
Επικοινωνία:

Re: Όρια με ολοκληρώματα

#162

Μη αναγνωσμένη δημοσίευση από Κοτρώνης Αναστάσιος » Κυρ Μαρ 20, 2011 10:50 pm

67) Ας υπολογισθεί, αν υπάρχει το όριο \displaystyle{\lim_{x\to+\infty}\sqrt{x}\int_{0}^{\pi/4}e^{x(\cos t-1)}\cos t\,dt}.


Εσύ....; Θα γίνεις κανίβαλος....;
Άβαταρ μέλους
Κοτρώνης Αναστάσιος
Επιμελητής
Δημοσιεύσεις: 3203
Εγγραφή: Κυρ Φεβ 22, 2009 11:11 pm
Τοποθεσία: Μπροστά στο πισί...
Επικοινωνία:

Re: Όρια με ολοκληρώματα

#163

Μη αναγνωσμένη δημοσίευση από Κοτρώνης Αναστάσιος » Δευ Μαρ 21, 2011 8:36 pm

68) Ας υπολογιστεί, αν υπάρχει, το όριο \displaystyle{\lim_{x\to+\infty}\left(x^{3/2}\int_{0}^{\pi/2}e^{-x\sin^2t}\,dt-\frac{\sqrt{\pi}x}{2}\right)}.
τελευταία επεξεργασία από Κοτρώνης Αναστάσιος σε Τρί Μαρ 22, 2011 3:07 am, έχει επεξεργασθεί 1 φορά συνολικά.


Εσύ....; Θα γίνεις κανίβαλος....;
nonlinear
Δημοσιεύσεις: 290
Εγγραφή: Σάβ Αύγ 28, 2010 3:51 am

Re: Όρια με ολοκληρώματα

#164

Μη αναγνωσμένη δημοσίευση από nonlinear » Δευ Μαρ 21, 2011 10:31 pm

69) Να υπολογιστεί το όριο :

\displaystyle{\mathop {\lim }\limits_{n \to \infty } \left( {\frac{1}{{\sqrt n }} \cdot \sqrt[{{n^2}}]{{\prod\limits_{i = 1}^n {{i^i}} }}} \right)}


Άβαταρ μέλους
Κοτρώνης Αναστάσιος
Επιμελητής
Δημοσιεύσεις: 3203
Εγγραφή: Κυρ Φεβ 22, 2009 11:11 pm
Τοποθεσία: Μπροστά στο πισί...
Επικοινωνία:

Re: Όρια με ολοκληρώματα

#165

Μη αναγνωσμένη δημοσίευση από Κοτρώνης Αναστάσιος » Τρί Μαρ 22, 2011 2:52 am

nonlinear έγραψε:69) Να υπολογιστεί το όριο :

\displaystyle{\mathop {\lim }\limits_{n \to \infty } \left( {\frac{1}{{\sqrt n }} \cdot \sqrt[{{n^2}}]{{\prod\limits_{i = 1}^n {{i^i}} }}} \right)}
\displaystyle{n^{-1/2}\left(\prod_{i=1}^{n}i^i\right)^{1/n^2}=\exp\left(-\frac{\ln n}{2}+\frac{1}{n}\sum_{i=1}^{n}\frac{i}{n}\ln\frac{i}{n}\cdot n\right)=\exp\left(-\frac{\ln n}{2}+\frac{1}{n}\sum_{i=1}^{n}\frac{i}{n}\ln\frac{i}{n}+\frac{\ln n}{n^2}\frac{(1+n)n}{2}\right)=}
\displaystyle{\sqrt[2n]{n}\exp\left(\frac{1}{n}\sum_{i=1}^{n}\frac{i}{n}\ln\frac{i}{n}\right)\to\exp\left(\int_{0}^{1}x\ln x\,dx\right)=e^{-1/4}}.


Εσύ....; Θα γίνεις κανίβαλος....;
nonlinear
Δημοσιεύσεις: 290
Εγγραφή: Σάβ Αύγ 28, 2010 3:51 am

Re: Όρια με ολοκληρώματα

#166

Μη αναγνωσμένη δημοσίευση από nonlinear » Τρί Μαρ 22, 2011 2:48 pm

Κοτρώνης Αναστάσιος έγραψε:
nonlinear έγραψε:69) Να υπολογιστεί το όριο :

\displaystyle{\mathop {\lim }\limits_{n \to \infty } \left( {\frac{1}{{\sqrt n }} \cdot \sqrt[{{n^2}}]{{\prod\limits_{i = 1}^n {{i^i}} }}} \right)}
\displaystyle{n^{-1/2}\left(\prod_{i=1}^{n}i^i\right)^{1/n^2}=\exp\left(-\frac{\ln n}{2}+\frac{1}{n}\sum_{i=1}^{n}\frac{i}{n}\ln\frac{i}{n}\cdot n\right)=\exp\left(-\frac{\ln n}{2}+\frac{1}{n}\sum_{i=1}^{n}\frac{i}{n}\ln\frac{i}{n}+\frac{\ln n}{n^2}\frac{(1+n)n}{2}\right)=}
\displaystyle{\sqrt[2n]{n}\exp\left(\frac{1}{n}\sum_{i=1}^{n}\frac{i}{n}\ln\frac{i}{n}\right)\to\exp\left(\int_{0}^{1}x\ln x\,dx\right)=e^{-1/4}}.
To \displaystyle{\int_{0}^{1}x\ln x\,dx\right)} δεν οριζεται οποτε παμε εψιλοντικα (lim ε->0). Το αποτελεσμα βεβαια δεν αλλαζει.


Ωmega Man
Δημοσιεύσεις: 1264
Εγγραφή: Παρ Ιουν 05, 2009 8:17 am

Re: Όρια με ολοκληρώματα

#167

Μη αναγνωσμένη δημοσίευση από Ωmega Man » Τετ Μαρ 23, 2011 3:34 am

Αυτό που λέει ο nonlinear λίγο διαφορετικά......(απλά για να υπάρχει).

\displaystyle{\bf \int_{0}^{1}x\log(x)\;\texttt{d}x=\lim_{n\rightarrow +\infty}\int_\frac{1}{n}^{1}x\log(x)\;\texttt{d}x} και το τελευταίο

\displaystyle{\bf \int_{\frac{1}{n}}^{1}x\log(x)\;\texttt{d}x=\frac{x^{2}}{2}\log(x)\bigg|_{\frac{1}{n}}^{1}-\int_{\frac{1}{n}}^{1}\frac{x}{2}\;\texttt{d}x=\frac{1}{2n^{2}}\log(n)-\frac{1}{2}\left(\frac{1}{2}-\frac{1}{2n^{2}}\right)=\frac{1}{2n^{2}}\log(n)+\frac{1}{4n^{2}}-\frac{1}{4}\overset{n\rightarrow+\infty}{\longrightarrow}-\frac{1}{4}} και έτσι \displaystyle{\bf \int_{0}^{1}x\log(x)\;\texttt{d}x=-\frac{1}{4}}.


What's wrong with a Greek in Hamburg?
Άβαταρ μέλους
Κοτρώνης Αναστάσιος
Επιμελητής
Δημοσιεύσεις: 3203
Εγγραφή: Κυρ Φεβ 22, 2009 11:11 pm
Τοποθεσία: Μπροστά στο πισί...
Επικοινωνία:

Re: Όρια με ολοκληρώματα

#168

Μη αναγνωσμένη δημοσίευση από Κοτρώνης Αναστάσιος » Δευ Μαρ 28, 2011 7:50 pm

70) Υπολογίστε, αν υπάρχει, το όριο \displaystyle{\lim_{x\to+\infty}\sqrt{x}\int_{-\infty}^{+\infty}e^{-x(t^2+1-\cos t)}\frac{t}{\sinh t}\,dt}.


Εσύ....; Θα γίνεις κανίβαλος....;
Άβαταρ μέλους
Σεραφείμ
Επιμελητής
Δημοσιεύσεις: 1872
Εγγραφή: Τετ Μάιος 20, 2009 9:14 am
Τοποθεσία: Θεσσαλονίκη - Γιάννενα

Re: Όρια με ολοκληρώματα

#169

Μη αναγνωσμένη δημοσίευση από Σεραφείμ » Κυρ Απρ 17, 2011 9:00 pm

Κοτρώνης Αναστάσιος έγραψε:67) Ας υπολογισθεί, αν υπάρχει το όριο \displaystyle{\lim_{x\to+\infty}\sqrt{x}\int_{0}^{\pi/4}e^{x(\cos t-1)}\cos t\,dt}.
Θα εφαρμόσουμε την μέθοδο του Laplace:
Έστω \displaystyle{x \to  + \infty } και s: θέση ολικού ελαχίστου της συνάρτησης g(t) , δηλαδή \displaystyle{g'\left( s \right) = 0{\text{  \&    }}g''\left( s \right) > 0} . Τότε \displaystyle{\int\limits_{ - \infty }^{ + \infty } {f\left( t \right){e^\big{{ - xg\left( t \right)}}}dt}  \sim \sqrt {\frac{{2\pi }}{{x \cdot g''\left( s \right)}}} f\left( s \right){e^\big{{ - xg\left( s \right)}}}} .

Η απόδειξή του, (χωρίς αυστηρότητα) ακολουθεί τα εξής: \displaystyle{\int\limits_{ - \infty }^{ + \infty } {f\left( t \right){e^\big{{ - xg\left( t \right)}}}dt}  \sim \int\limits_{ - \infty }^{ + \infty } {f\left( t \right){e^\big{{ - x\left( {g\left( s \right) + \frac{1}{2}g''\left( s \right){{\left( {t - s} \right)}^2} + ..} \right)}}}dt}  \sim f\left( s \right){e^\big{{ - xg\left( s \right)}}}\int\limits_{ - \infty }^{ + \infty } {{e^\big{{ - \dfrac{x}{2}g''\left( s \right){y^2}}}}dy}  \sim }

\displaystyle{ \sim f\left( s \right){e^\big{{ - xg\left( s \right)}}}\int\limits_{ - \infty }^{ + \infty } {{e^{ - {{\left( {\sqrt {\dfrac{{xg''\left( s \right)}}{2}} y} \right)}^2}}}dy}  \sim \sqrt {\frac{{2\pi }}{{xg''\left( s \right)}}} f\left( s \right){e^\big{{ - xg\left( s \right)}}}} http://www2.ph.ed.ac.uk/~dmarendu/MOMP/lecture04.pdf

Στο θέμα μας.

Για \displaystyle{f\left( t \right) = \cos t{\text{ }}{\text{,  }}g\left( t \right) = 1 - \cos t} και επειδή στην θέση t=0 έχουμε ολικό ελάχιστο για την g(t) με \displaystyle{g'\left( 0 \right) = 0{\text{  \&    }}g''\left( 0 \right) = 1} ,

για «μεγάλα» x θα έχουμε \displaystyle{I = \int\limits_0^{\pi /4} {{e^\big{{ - x\left( {1 - \cos \left( t \right)} \right)}}}\cos \left( t \right)dt}  = \int\limits_0^{\pi /4} {{e^\big{{ - x\left( {\dfrac{{{t^2}}}{2} - \dfrac{{{t^4}}}{{4!}} + ..} \right)}}}\cos \left( t \right)dt}  \sim \int\limits_0^{\pi /4} {{e^\big{{ - {{\left( {\dfrac{{t\sqrt x }}{{\sqrt 2 }}} \right)}^2}}}}\cos \left( t \right)dt}  = \mathop  = \limits^\big{{\dfrac{{t\sqrt x }}{{\sqrt 2 }} = y}}  = }

\displaystyle{ = \frac{{\sqrt 2 }}{{\sqrt x }}\int\limits_0^{\pi \sqrt x /4\sqrt 2 } {{e^{ - {y^2}}}\cos \left( {y\frac{{\sqrt 2 }}{{\sqrt x }}} \right)dy}  \sim \frac{{\sqrt 2 }}{{\sqrt x }}\int\limits_0^\infty  {{e^\big{{ - {y^2}}}}dy}  = \frac{{\sqrt 2 }}{{\sqrt x }} \cdot \frac{{\sqrt \pi  }}{2}}

Τότε \displaystyle{\mathop {\lim }\limits_{x \to \infty } \sqrt x \int\limits_0^{\pi /4} {{e^\big{{ - x\left( {1 - \cos \left( t \right)} \right)}}}\cos \left( t \right)dt}  = \mathop {\lim }\limits_{x \to \infty } \sqrt x \frac{{\sqrt 2 }}{{\sqrt x }} \cdot \frac{{\sqrt \pi  }}{2} = \sqrt {\frac{\pi }{2}} } .


Σεραφείμ Τσιπέλης
Άβαταρ μέλους
Κοτρώνης Αναστάσιος
Επιμελητής
Δημοσιεύσεις: 3203
Εγγραφή: Κυρ Φεβ 22, 2009 11:11 pm
Τοποθεσία: Μπροστά στο πισί...
Επικοινωνία:

Re: Όρια με ολοκληρώματα

#170

Μη αναγνωσμένη δημοσίευση από Κοτρώνης Αναστάσιος » Δευ Απρ 18, 2011 11:46 am

Όμορφα! Αυτό ήταν το κλού της άσκησης Σεραφείμ.

Μια λεπτομερή απόδειξη της μεθόδου του Laplace, καθώς και μια διαφορετική αντιμετώπιση του θέματος με χρήση του Λήμματος του Watson (καθώς και την απόδειξή του) μπορεί κανείς να δει στα Θέματα Ανάλυσης.


Εσύ....; Θα γίνεις κανίβαλος....;
vzf
Δημοσιεύσεις: 310
Εγγραφή: Κυρ Φεβ 28, 2010 11:11 pm

Re: Όρια με ολοκληρώματα

#171

Μη αναγνωσμένη δημοσίευση από vzf » Τετ Μάιος 11, 2011 2:24 am

71)Yπολογίστε το όριο \displaystyle\lim_{n\rightarrow\infty}n\int_{0}^{\frac{\displaystyle\pi}{2}}{\left(1-\sqrt[n]{sinx}\right)dx}.


Άβαταρ μέλους
Κοτρώνης Αναστάσιος
Επιμελητής
Δημοσιεύσεις: 3203
Εγγραφή: Κυρ Φεβ 22, 2009 11:11 pm
Τοποθεσία: Μπροστά στο πισί...
Επικοινωνία:

Re: Όρια με ολοκληρώματα

#172

Μη αναγνωσμένη δημοσίευση από Κοτρώνης Αναστάσιος » Τετ Μάιος 11, 2011 9:38 pm

vzf έγραψε:71)Yπολογίστε το όριο \displaystyle\lim_{n\rightarrow\infty}n\int_{0}^{\frac{\displaystyle\pi}{2}}{\left(1-\sqrt[n]{\sin x}\right)dx:=\lim_{n\to+\infty}\int_{0}^{\pi/2}f_{n}(x)\,dx}.
Θεωρούμε τη συνάρτηση f(t)=t\sin ^{1/t}x όπου x\in(0,\pi/2] τυχαίο αλλά σταθερό και t\geq1.

Είναι f{'}(t)=\sin^{1/t}x\left(1-\frac{\ln(\sin x)}{t}\right)>0, άρα n_{1}<n_{2}\Rightarrow n_{1}(1-\sin^{1/n_{1}}x)<n_{2}(1-\sin^{1/n_{2}}x)\Rightarrow f_{n_{1}}(x)<f_{n_{2}}(x).

Επίσης είναι f_{n}(x)\stackrel{n\to+\infty}{\longrightarrow}-\ln(\sin x) για x\in(0,\pi/2], οπότε από το θεώρημα μονότονης σύγκλισης είναι \displaystyle{\lim_{n\to+\infty}\int_{0}^{\pi/2}f_{n}(x)\,dx=\int_{0}^{\pi/2}-\ln(\sin x)\,dx}, το οποίο έχουμε ξανασυζητήσει σε παλαιότερο ποστ ότι κάνει \displaystyle{\frac{\pi\ln2}{2}}.


Εσύ....; Θα γίνεις κανίβαλος....;
Άβαταρ μέλους
mathxl
Δημοσιεύσεις: 6736
Εγγραφή: Τρί Δεκ 23, 2008 3:49 pm
Τοποθεσία: Σιδηρόκαστρο
Επικοινωνία:

Re: Όρια με ολοκληρώματα

#173

Μη αναγνωσμένη δημοσίευση από mathxl » Παρ Μάιος 27, 2011 1:59 pm

72) Να υπολογίσετε το όριο
\displaystyle{\mathop {\lim }\limits_{n \to \infty } \int_a^b {\frac{{si{n^n}x}}{{si{n^n}x + co{s^n}x}}} dx}

όταν \displaystyle{0 \le a < b \le \frac{\pi }{2}}


Ποτε δεν κάνω λάθος! Μια φορά νομιζα πως είχα κάνει, αλλά τελικά έκανα λάθος!
Απ' τα τσακάλια δεν γλυτώνεις μ' ευχές η παρακάλια. Κ. Βάρναλης
Aπέναντι στις αξίες σου να είσαι ανυποχώρητος

Ενεργό μέλος από 23-12-2008 ως και 17-8-2014 (δεν θα απαντήσω σε πμ)
Άβαταρ μέλους
mathxl
Δημοσιεύσεις: 6736
Εγγραφή: Τρί Δεκ 23, 2008 3:49 pm
Τοποθεσία: Σιδηρόκαστρο
Επικοινωνία:

Re: Όρια με ολοκληρώματα

#174

Μη αναγνωσμένη δημοσίευση από mathxl » Δευ Μάιος 30, 2011 9:24 pm

73)
\displaystyle\lim_{n\rightarrow\infty}\int_\pi^{2\pi}\frac{|sinnx+cosnx|}{x}dx


Ποτε δεν κάνω λάθος! Μια φορά νομιζα πως είχα κάνει, αλλά τελικά έκανα λάθος!
Απ' τα τσακάλια δεν γλυτώνεις μ' ευχές η παρακάλια. Κ. Βάρναλης
Aπέναντι στις αξίες σου να είσαι ανυποχώρητος

Ενεργό μέλος από 23-12-2008 ως και 17-8-2014 (δεν θα απαντήσω σε πμ)
Άβαταρ μέλους
Κοτρώνης Αναστάσιος
Επιμελητής
Δημοσιεύσεις: 3203
Εγγραφή: Κυρ Φεβ 22, 2009 11:11 pm
Τοποθεσία: Μπροστά στο πισί...
Επικοινωνία:

Re: Όρια με ολοκληρώματα

#175

Μη αναγνωσμένη δημοσίευση από Κοτρώνης Αναστάσιος » Τρί Αύγ 09, 2011 6:49 pm

Κοτρώνης Αναστάσιος έγραψε:\displaystyle{61)} Υπολογισθήτω το \displaystyle{\underset{n\to \infty }{\mathop{\lim }}\,\int_{-\pi }^{\pi }{\arctan \left( \frac{1}{n}\sum\limits_{k=1}^{n}{k{{\sin }^{2}}(kx)} \right)\,dx}}.
Εδώ.


Εσύ....; Θα γίνεις κανίβαλος....;
socrates
Επιμελητής
Δημοσιεύσεις: 6461
Εγγραφή: Δευ Μαρ 09, 2009 1:47 pm
Τοποθεσία: Θεσσαλονίκη
Επικοινωνία:

Re: Όρια με ολοκληρώματα

#176

Μη αναγνωσμένη δημοσίευση από socrates » Σάβ Οκτ 15, 2011 2:35 pm



Θανάσης Κοντογεώργης
Άβαταρ μέλους
Κοτρώνης Αναστάσιος
Επιμελητής
Δημοσιεύσεις: 3203
Εγγραφή: Κυρ Φεβ 22, 2009 11:11 pm
Τοποθεσία: Μπροστά στο πισί...
Επικοινωνία:

Re: Όρια με ολοκληρώματα

#177

Μη αναγνωσμένη δημοσίευση από Κοτρώνης Αναστάσιος » Σάβ Οκτ 15, 2011 4:13 pm

Από το ανάπτυγμα Taylor της \tan t στο 0, βλέπουμε ότι \displaystyle{\cot t^2=\frac{1}{t^2}+\mathcal O(t^2)} άρα η \cot t^2 συμπεριφέρεται κοντά στο 0 όπως η \displaystyle{\frac{1}{t^2}}. Κάνουμε λοιπόν το εξής για να απλουστεύσουμε την κατάσταση:

Γράφουμε \displaystyle{\int_{\frac {1}{x+1}}^{\frac{1}{x}}\cot t^{2}\ \text{d}t =\int_{\frac {1}{x+1}}^{\frac{1}{x}}\cot t^{2}-\frac{1}{t^2}\,dt+\int_{\frac {1}{x+1}}^{\frac{1}{x}}\frac{1}{t^2}\,dt }.

Τώρα \displaystyle{\cot t^{2}-\frac{1}{t^2}=\mathcal O(t^2)}, άρα \displaystyle{\int_{\frac {1}{x+1}}^{\frac{1}{x}}\cot t^{2}-\frac{1}{t^2}\,dt=\mathcal O\left(\int_{1/x}^{1/(x+1)}t^2\right)=\mathcal O\left(\frac{1}{(x+1)^3}-\frac{1}{x^3}\right)=\mathcal O(x^{-4})} ενώ

\displaystyle{\int_{\frac {1}{x+1}}^{\frac{1}{x}}\frac{1}{t^2}\,dt=1}, άρα \displaystyle{\int_{\frac {1}{x+1}}^{\frac{1}{x}}\cot t^{2}\ \text{d}t =1+\mathcal O(x^{-4})\to1}.

Μπορούμε να βρούμε και καλύτερη προσσέγγιση της ποσότητας ξεκινώντας από καλύτερη προσσέγγιση της \displaystyle{\tan t} κοντά στο 0.


Εσύ....; Θα γίνεις κανίβαλος....;
Άβαταρ μέλους
Κοτρώνης Αναστάσιος
Επιμελητής
Δημοσιεύσεις: 3203
Εγγραφή: Κυρ Φεβ 22, 2009 11:11 pm
Τοποθεσία: Μπροστά στο πισί...
Επικοινωνία:

Re: Όρια με ολοκληρώματα

#178

Μη αναγνωσμένη δημοσίευση από Κοτρώνης Αναστάσιος » Πέμ Νοέμ 03, 2011 4:06 pm

75) Αν a>0, ας βρεθεί αν υπάρχει το \displaystyle{\lim_{n\to+\infty}\left(\int_{0}^{1}(1+ax^n)^n\,dx\right)^{1/n}}


Εσύ....; Θα γίνεις κανίβαλος....;
vzf
Δημοσιεύσεις: 310
Εγγραφή: Κυρ Φεβ 28, 2010 11:11 pm

Re: Όρια με ολοκληρώματα

#179

Μη αναγνωσμένη δημοσίευση από vzf » Κυρ Φεβ 05, 2012 8:26 am

\displaystyle{76)} Να υπολογίσετε το όριο \displaystyle{\displaystyle \lim_{{{t}\to{0}}}\frac{{1}}{{t}}{\int_{{0}}^{{t}}}{\left|{\sin{{\left(\frac{{1}}{{x}}\right)}}}\right|}{\left.{d}{x}\right}}.

\displaystyle{77)} Να υπολογίσετε το όριο \displaystyle{\displaystyle \lim_{{{t}\to{0}}}\frac{{1}}{{t}}{\int_{{0}}^{{t}}}{\cos{{\left({\sin{{\left({\tan{{\left(\frac{{1}}{{x}}\right)}}}\right)}}}\right)}}}{\left.{d}{x}\right}}.


Άβαταρ μέλους
Κοτρώνης Αναστάσιος
Επιμελητής
Δημοσιεύσεις: 3203
Εγγραφή: Κυρ Φεβ 22, 2009 11:11 pm
Τοποθεσία: Μπροστά στο πισί...
Επικοινωνία:

Re: Όρια με ολοκληρώματα

#180

Μη αναγνωσμένη δημοσίευση από Κοτρώνης Αναστάσιος » Κυρ Φεβ 05, 2012 3:10 pm

vzf έγραψε:\displaystyle{76)} Να υπολογίσετε το όριο \displaystyle{\displaystyle \lim_{{{t}\to{0}}}\frac{{1}}{{t}}{\int_{{0}}^{{t}}}{\left|{\sin{{\left(\frac{{1}}{{x}}\right)}}}\right|}{\left.{d}{x}\right}}.
Για το δεξί πλευρικό :

Κάνοντας την αλλαγή μεταβλητής u=1/t στο όριο και την 1/x=y στο ολοκλήρωμα, ζητάμε να βρούμε το

\displaystyle{\lim_{u\to+\infty}u\int_{u}^{+\infty}\frac{|\sin y|}{y^2}\,dy}.

Η |\sin y| ως συνεχής στο \mathbb R θα έχει αρχική έστω F με F'=|\sin x| για x\in\mathbb R. Αυτή η F θα είναι επίσης φραγμένη.

Τότε με ολοκλήρωση κατά μέρη έχουμε

\displaystyle{\begin{aligned}u\int_{u}^{+\infty}\frac{|\sin y|}{y^2}\,dy&=u\left(\frac{F(y)}{y^2}\Bigg|_{u}^{+\infty}+2\int_{u}^{+\infty}\frac{F(y)}{y^3}\,dy\right) \notag \\ &\stackrel{*}{=}\frac{F(u)}{u}+2u\int_{u}^{+\infty}\frac{F(y)}{y^3}\,dy\end{aligned}}

* Η F φραγμένη.

Τώρα \displaystyle{\frac{F(u)}{u}\to0} πάλι διότι η F είναι φραγμένη και για τον ίδιο πάλι λόγο

\displaystyle{\Bigg|u\int_{u}^{+\infty}\frac{F(y)}{y^3}\,dy\Bigg|\leq u\int_{u}^{+\infty}\Bigg|\frac{F(y)}{y^3}\Bigg|\,dy\leq\frac{c}{u}\to0}, άρα το δεξί πλευρικό είναι 0.

Για το αριστερό πλευρικό κάνουμε τα αντίστοιχα.


Εσύ....; Θα γίνεις κανίβαλος....;
Απάντηση

Επιστροφή σε “ΑΝΑΛΥΣΗ”

Μέλη σε σύνδεση

Μέλη σε αυτήν τη Δ. Συζήτηση: Δεν υπάρχουν εγγεγραμμένα μέλη και 13 επισκέπτες